http://7sage.com/lsat_explanations/lsat-56-section-3-question-20/
The ... (D) without hesitation. The question stem asks "which one of ... heart disease. Furthermore, the question stem asks for "most weaken ... to (B) given the question stem?
I' ... very confused with this specific question, one of the few ... . It's a weaken question but the correct answer choice ... think to deeply into this question and miss the obvious ...
http://7sage.com/lsat_explanations/lsat-26-section-3-question-24/
This question is beyond sucky. Did anyone else have any problems with determining the right answer for this question? Thanks.